Đến nội dung

Hình ảnh

bai gioi han hay


  • Please log in to reply
Chủ đề này có 26 trả lời

#21
AnSatTruyHinh

AnSatTruyHinh

    Binh nhất

  • Thành viên
  • 45 Bài viết

Công thứ giwois hạn của tổng bằng tổng các giới hạn thì đúng nhưng phải thay trong cùng mộy quá trình bạn ah!

Bạn thử cho mình một phản ví dụ đi.nếu f(x) và g(x) có lim hữu hạn thì hoàn toàn có ct đó (thực chất chỉ là chưa đưa ra ngoài lim mà thôi)

#22
inhtoan

inhtoan

    <^_^)

  • Thành viên
  • 964 Bài viết

Bạn thử cho mình một phản ví dụ đi.nếu f(x) và g(x) có lim hữu hạn thì hoàn toàn có ct đó (thực chất chỉ là chưa đưa ra ngoài lim mà thôi)

Bạn AnSatTruyHinh xem ví dụ sau...

Vậy mình thử lấy 1 ví dụ xem theo bạn cách sau có hợp lí không nhé.
Tìm $\mathop {\lim }\limits_{x \to + \infty } (\sqrt {{x^2} - 2x + 3} - x)$
Lời giải.
Ta có $L = \mathop {\lim }\limits_{x \to + \infty } (\sqrt {{x^2} - 2x + 3} - x) = \mathop {\lim }\limits_{x \to + \infty } \left( {x\sqrt {1 - \dfrac{2}{x} + \dfrac{3}{{{x^2}}}} - x} \right)$
Vì $\mathop {\lim }\limits_{x \to + \infty } \sqrt {1 - \dfrac{2}{x} + \dfrac{3}{{{x^2}}}} = 1$
nên $L = \mathop {\lim }\limits_{x \to + \infty } (x - x) = 0.$



#23
AnSatTruyHinh

AnSatTruyHinh

    Binh nhất

  • Thành viên
  • 45 Bài viết

Bạn AnSatTruyHinh xem ví dụ sau...


Mình chưa thấy gì mâu thuẫn với lời mình nói cả.Với lại lời giải đó sai ở chỗ $\lim_{x \to +\infty}{x.\sqrt{1-\dfrac{2}{x}+\dfrac{3}{x^2}}=\lim_{x \to +\infty}{x}$ vì $\lim_{x \to +\infty}{x}=+\infty$ nên không áp dụng dc ct tổng hai lim và tích hai lim,cùng dạng sai với bài trên...

#24
CD13

CD13

    Thượng úy

  • Thành viên
  • 1456 Bài viết

$L = \mathop {\lim }\limits_{x \to + \infty } (x - x) .$
Dạng $\infty - \infty$



#25
inhtoan

inhtoan

    <^_^)

  • Thành viên
  • 964 Bài viết

Mình chưa thấy gì mâu thuẫn với lời mình nói cả.Với lại lời giải đó sai ở chỗ $\lim_{x \to +\infty}{x.\sqrt{1-\dfrac{2}{x}+\dfrac{3}{x^2}}=\lim_{x \to +\infty}{x}$ vì $\lim_{x \to +\infty}{x}=+\infty$ nên không áp dụng dc ct tổng hai lim và tích hai lim,cùng dạng sai với bài trên...

Đây là 1 ví dụ mình lấy từ phần "sai ở đâu, sửa cho đúng" trên báo THTT (tuyển tập quyển 2). Mình xin ghi lại lời giải thích trong sách (Sorry chủ topic vì post bài không đúng chủ đề lắm).

Tìm $\mathop {\lim }\limits_{x \to + \infty } (\sqrt {{x^2} - 2x + 3} - x)$
Lời giải.
Ta có $L = \mathop {\lim }\limits_{x \to + \infty } (\sqrt {{x^2} - 2x + 3} - x) = \mathop {\lim }\limits_{x \to + \infty } \left( {x\sqrt {1 - \dfrac{2}{x} + \dfrac{3}{{{x^2}}}} - x} \right)$
Vì $\mathop {\lim }\limits_{x \to + \infty } \sqrt {1 - \dfrac{2}{x} + \dfrac{3}{{{x^2}}}} = 1$
nên $L = \mathop {\lim }\limits_{x \to + \infty } (x - x) = 0.$


Ai cũng biết là lời giải trên sai, nhưng sai ở đâu ? Nhiều bạn lí giải theo nhiều cách khác nhau.

Chẳng hạn:
1) Viết $\mathop {\lim }\limits_{x \to + \infty } (x - x) = 0$ là sai (!). Oan quá ! Viết như vậy quá đúng ! Lưu ý cho là $\mathop {\lim }\limits_{x \to + \infty } (x - x) = \mathop {\lim }\limits_{x \to + \infty } 0 = 0$.

2) Thực ra $\mathop {\lim }\limits_{x \to + \infty } (x - x) = \mathop {\lim }\limits_{x \to + \infty } x - \mathop {\lim }\limits_{x \to + \infty } x = ( + \infty ) - ( + \infty )$ chưa chắc đã bằng 0 (!).
Phân tích này bị sai ngay khi viết $\mathop {\lim }\limits_{x \to + \infty } (x - x) = \mathop {\lim }\limits_{x \to + \infty } x - \mathop {\lim }\limits_{x \to + \infty } x$. Hình như cái sai này cũng chẳng kém gì cái sai của cách giải ở trên.

Xin nếu ra "căn bệnh" của lời giải trên.
Không thể hiểu rằng ( chứ không phải "không đúng" như bạn AnSatTruyHinh đã nói)
$\mathop {\lim }\limits_{x \to + \infty } \left( {x\sqrt {1 - \dfrac{2}{x} + \dfrac{3}{{{x^2}}}} } \right) = x$ mặc dù $\mathop {\lim }\limits_{x \to + \infty } \sqrt {1 - \dfrac{2}{x} + \dfrac{3}{{{x^2}}}} = 1$.
Chính vì vậy, không thể có đẳng thức $\mathop {L = \lim (x - x)}\limits_{x \to + \infty }$. Mắt xích của lời giải "bị tuột" là ở đó.


p/s: Thông qua ví dụ này mong bạn AnSatTruyHinh nên xem xét lại ý kiến của bạn.Nếu có một quy tắc tìm giới hạn nào đúng như bạn nói thì bạn cho mình xin nguồn hoặc bạn có thể chứng minh quy tắc đó (chứ đừng dựa vào một vài trường hợp đặc biệt).
Xin chú ý rằng nếu có quy tắc tính giới hạn của tích 2 hàm số đúng như bạn nói thì trong SGK đã không phải phân ra làm 2 phần:
Với $\mathop {\lim }\limits_{x \to {x_0}} f(x) = L;\mathop {\lim }\limits_{x \to {x_0}} g(x) = M.(M,L \in {\mkern 1mu} {\rm{R}})$ thì
$1)\mathop {\lim }\limits_{x \to {x_0}} [f(x).g(x)] = L.M.$
2) Đặc biệt với c là một hằng số thì $\mathop {\lim }\limits_{x \to {x_0}} [cf(x)] = cL.$

Bài viết đã được chỉnh sửa nội dung bởi inhtoan: 21-10-2010 - 12:44


#26
AnSatTruyHinh

AnSatTruyHinh

    Binh nhất

  • Thành viên
  • 45 Bài viết

Đây là 1 ví dụ mình lấy từ phần "sai ở đâu, sửa cho đúng" trên báo THTT (tuyển tập quyển 2). Mình xin ghi lại lời giải thích trong sách (Sorry chủ topic vì post bài không đúng chủ đề lắm).

Ai cũng biết là lời giải trên sai, nhưng sai ở đâu ? Nhiều bạn lí giải theo nhiều cách khác nhau.

Chẳng hạn:
1) Viết $\mathop {\lim }\limits_{x \to + \infty } (x - x) = 0$ là sai (!). Oan quá ! Viết như vậy quá đúng ! Lưu ý cho là $\mathop {\lim }\limits_{x \to + \infty } (x - x) = \mathop {\lim }\limits_{x \to + \infty } 0 = 0$.

2) Thực ra $\mathop {\lim }\limits_{x \to + \infty } (x - x) = \mathop {\lim }\limits_{x \to + \infty } x - \mathop {\lim }\limits_{x \to + \infty } x = ( + \infty ) - ( + \infty )$ chưa chắc đã bằng 0 (!).
Phân tích này bị sai ngay khi viết $\mathop {\lim }\limits_{x \to + \infty } (x - x) = \mathop {\lim }\limits_{x \to + \infty } x - \mathop {\lim }\limits_{x \to + \infty } x$. Hình như cái sai này cũng chẳng kém gì cái sai của cách giải ở trên.

Xin nếu ra "căn bệnh" của lời giải trên.
Không thể hiểu rằng ( chứ không phải "không đúng" như bạn AnSatTruyHinh đã nói)
$\mathop {\lim }\limits_{x \to + \infty } \left( {x\sqrt {1 - \dfrac{2}{x} + \dfrac{3}{{{x^2}}}} } \right) = x$ mặc dù $\mathop {\lim }\limits_{x \to + \infty } \sqrt {1 - \dfrac{2}{x} + \dfrac{3}{{{x^2}}}} = 1$.
Chính vì vậy, không thể có đẳng thức $\mathop {L = \lim (x - x)}\limits_{x \to + \infty }$. Mắt xích của lời giải "bị tuột" là ở đó.


p/s: Thông qua ví dụ này mong bạn AnSatTruyHinh nên xem xét lại ý kiến của bạn.Nếu có một quy tắc tìm giới hạn nào đúng như bạn nói thì bạn cho mình xin nguồn hoặc bạn có thể chứng minh quy tắc đó (chứ đừng dựa vào một vài trường hợp đặc biệt).
Xin chú ý rằng nếu có quy tắc tính giới hạn của tích 2 hàm số đúng như bạn nói thì trong SGK đã không phải phân ra làm 2 phần:
Với $\mathop {\lim }\limits_{x \to {x_0}} f(x) = L;\mathop {\lim }\limits_{x \to {x_0}} g(x) = M.(M,L \in {\mkern 1mu} {\rm{R}})$ thì
$1)\mathop {\lim }\limits_{x \to {x_0}} [f(x).g(x)] = L.M.$
2) Đặc biệt với c là một hằng số thì $\mathop {\lim }\limits_{x \to {x_0}} [cf(x)] = cL.$

???
Hình như bạn inhtoan không hiểu mình nói thì phải.Bài đó rõ ràng "không thể hiểu"=" không đúng".Công thức Tích,tổng,thương hai lim rõ ràng cần có giới hạn hữu hạn (TH kq là vô cùng đã có hẳn một bài trong SGK về QUY TẮC DẤU).Còn quy tắc chính là kq trong sách giáo khoa lim(f+g)=M+N,lim(f-M)=0<-->lim f=M
Bài giả trên và cả của bạn ongtroi khi dùng ct tìm lim đều phạm điều kiện.mình không muốn tranh cãi thêm nữa về vấn đề này,bạn không tin có thể hỏi các thầy cô.Chấm.

#27
khacduongpro_165

khacduongpro_165

    Thiếu úy

  • Thành viên
  • 594 Bài viết

???
Hình như bạn inhtoan không hiểu mình nói thì phải.Bài đó rõ ràng "không thể hiểu"=" không đúng".Công thức Tích,tổng,thương hai lim rõ ràng cần có giới hạn hữu hạn (TH kq là vô cùng đã có hẳn một bài trong SGK về QUY TẮC DẤU).Còn quy tắc chính là kq trong sách giáo khoa lim(f+g)=M+N,lim(f-M)=0<-->lim f=M
Bài giả trên và cả của bạn ongtroi khi dùng ct tìm lim đều phạm điều kiện.mình không muốn tranh cãi thêm nữa về vấn đề này,bạn không tin có thể hỏi các thầy cô.Chấm.

Tớ Post bài này lên là xem các bạn trên diễn đàn giải thế nào. Bởi vì thực tế thì sự tranh cãi đó tớ cũng đang gặp pahir!
"Phong độ là nhất thời, đẳng cấp là mãi mãi"!!!




0 người đang xem chủ đề

0 thành viên, 0 khách, 0 thành viên ẩn danh